PT17.S3.Q08 - political advocate: campaigns for elective

Giant PandaGiant Panda Alum Member
edited January 2017 in Logical Reasoning 274 karma
Hi Guys,

Can you guys please help me take a look if my analysis on B & C is correct? https://7sage.com/lsat_explanations/lsat-17-section-3-question-08/


The question is very much like a SA question. The answer choices can be quickly eliminated by a match principle into the sufficient condition. However, I think we can expand on this problem more.

A is correct. So won't go into detail about it.

B. The sentence is wrong based on 2 reasons. The first reason is by putting the conclusion as the sufficient condition. Even if we were to change the answer into: If election campaigns are to be funded from public funds, it will allow politian to devote less time asking for money than serving the interest of the public, this is still wrong because it is formulated into a C-->P Relationship, while what we are looking for is P-->C relationship.

C is wrong because it talks of a different set that we do not know. Had this question be translated into an Inference MBT Except question, the asnwer choice then is correct.

D. is wrong based on 2 counts. The first count is of the same reason as B by messing up the location of the conclusion into the suffcient condition. But in addition to it that the question steam mix in an unrealted element. Evne if we have deleted that related element from the sentence, it is still incorrect and not 50% correct due to the location issue with the conclusion statement.

If we were to extent this answer choice's analysis a bit further.

Suppose that in this case, the answer choice for A is wrong too. But D is formulated in the following fashion: "if public funding of some activity produces a benefit to the public but also inevitably a special benefit for specific individuals, the activity should be fully funded by the public while the special interest taker group contributes proportionally more".

In this answer choice, iff answer choice is made wrong in other fashion. D will be correct on 2 counts. 1) The sentence is 50% correct. Although it do have irrelevant items in it but part of the argument goes through along with our principle. 2) It is the most correct answer in comparison with the rest of the answer choice.

Had LSAT do this, then the question's difficulty is pushed to a level 4 or 5 difficulty.

E. is wrong because it like C speaks of another different set of population that is above this univerise. Again, it can become a correct answer choice if it is an MBT Inference Except question.
Sign In or Register to comment.